Jillian calculates that see will take 95 minutes to run 7 miles. She runs the distance in 80 minutes. What is Jillian’s percent error? ​

Answers

Answer 1

Answer:

Kawaii, the problem is the comparison with the calculated time for Jillian and the actual time for Jillian to run the 7 miles.

 

%error=(calculated value - experiemental value)/calculated * 100%

 

%error=(95-80)/95*100%=15/95 *100% = 15.79 ≈ 16% error  

 

Note: 16% error is the approximate error because the numbers have 2 significant figures stated in the problem.

Step-by-step explanation:


Related Questions

Is the relation a function?
{(-6, -1), (5,-1), (0, -1), (-2, -1), (3, -1)}

Answers

Answer:

yes it is a function

Step-by-step explanation:

For that no two x values are the same with different y values

Becky made 4 quarts of chicken noodle soup in a big pot, then served 112-cup bowls of soup to 6 people. How much soup is left in the pot?

Answers

Answer:

1qt 3c

Step-by-step explanation:

00:00
Brice is finding the sum of 468 and 241 by breaking it into smaller problems
He uses place value and finds the sums of the hundreds, tens, and ones.
What is the sum of the tens? Enter your answer in the box.
1​

Answers

468+241=709 so 0 is in the tenth place because 6+4= 10 and one gets carried over to the 4+2

A. 256
B. 265
C. 297
D. 279​

Answers

Answer:

A.

Step-by-step explanation:

So calculate one of the triangles.

[tex]\frac{12*8}{2} =48[/tex]

48 * 4 = 192

192 + 64 = 256

What is -6 divided by -1.2?

Answers

Answer:

5.

Step-by-step explanation:

This is the same as saying:

-6/(-6/5).

To solve the equation, we multiply by the reciprocal and simplify. In this case, we multiply both sides of the equation by -5/6.

-6 * (-5/6) = 5.

The answer must be positive, as two negatives divided equal a positive.

I hope this helps!

The solution of expression after divide is,

⇒ 5

We have to given that;

⇒ - 6 is divided by - 1.2

Now, We can simplify as,

⇒ - 6 is divided by - 1.2

It can be written as,

⇒ - 6 ÷ - 1.2

⇒ - 6 / - 1.2

⇒ 6 / 1.2

⇒ 60 / 12

⇒ 5

Thus, The solution of expression after divide is,

⇒ 5

Learn more about the divide visit:

https://brainly.com/question/28119824

#SPJ6

What is the midpoint of the segment below? (2,3)(-3,-2)

Answers

Answer:

(-0.5, 0.5)

Step-by-step explanation:

If those two points are endpoints, just use the midpoint formula.

(-0.5, 0.5)

Answer:

(-1/2,1/2)

Step-by-step explanation:

To find the midpoint, add the x coordinates and divide by 2

(2+-3)/2 = -1/2

add the y coordinates and divide by 2

(3+-2)/2 = 1/2

The midpoint is (-1/2,1/2)

Line I and h intersect at what point

Answers

Answer:

please show where the lines are graphed in a picture

Step-by-step explanation:

HELP ASAP it says my questions should be at least 20 characters long so what I’m saying right now doesn’t mean anything.

Answers

Answer: See pic above for answer. I got it from Photomath

Plz put brainliest

Find the value of x in the triangle pair below

Answers

Answer:

x=16.97056274... or 12[tex]\sqrt{2}[/tex]

Step-by-step explanation:

the hypotenuse of a 45, 45, 90 triangle is the length of the leg times the square root of 2

a) the quadratic x^2-4x-21 can be written in the form (x+a)^2+b find the values of and and b

b) hence, or otherwise, solve the equation x^2-4x-21=0

Answers

Part a. A=-2. B=-25
Part b solutions; x=7 x=-3

Micheal has 1/2 of a yard of fabric to make book covers. Each book cover is made from 1/8 of a yard of fabric. How many book covers can Micheal make?

Answers

Answer:

4

Step-by-step explanation:

[tex]\frac{1}{2}[/tex] ÷ [tex]\frac{1}{8}[/tex] = 4

If f(x) = 2x + 1 and g(x) = x-2 what is the value of f(g(f(3)))? This is so confusing plz help me will mark brainliest A) 1 B) 3 C) 5 D) 7 E) 11

Answers

Answer:

11

Step-by-step explanation:

f(3) = 2*3+1 = 6+1 = 7

Then find g(7)

g(7) = 7-2 = 5

Then find f(5)

f(5) = 2*5 +1 = 10+1 = 11

f(g(f(3))) = 11

Answer:

11

Step-by-step explanation:

f(x)=2x + 1

f(3)= 2.3 + 1

f(3)=7

f(g(f(3))) = f(g(7))

Now, we have to find g(7)

g(x)=x -2

g(7)= 7 -2

g(7)=5

f(g(f(3))) = f(g(7)) = f(5)

now we have to find f(5)

f(x)=2x + 1

f(5)=2.5 + 1

f(5)=11

Hope this helps ^-^

BEST ANSWER GETS TO CHOOSE BRAINLESST OR FOLLOW!​

Answers

Answer:13 is>

14 is =

15 is >

16 is<

17 is<

18 is =

Step-by-step explanation:

I just know plz mark brainliest

evaluate the following expression -7x(7+9)

Answers

Answer:

-112

Step-by-step explanation:

Add

-7(7+9)

Multiply

-7x16

Wallah! You have the answer

-112

[tex]\text{Simplify the expression:}\\\\-7x(7+9)\\\\\text{Use the distributive property}\\\\-49x-63x\\\\\text{Combine like terms}\\\\\boxed{-112x}[/tex]

What is the equation in slope-intercept form of a line with slope of 5 and y-intercept of 3?

Answers

y=5x+3

Because the starting value(y intercept is 3) and the slope(x) is 5

Pls help me on this................

Answers

Answer:

Step-by-step explanation:

Find the solution(s) to x^2- 14x + 49 = 0.
O A. x=-2 and x = 7
B. x= -1 and x = 14
C. x= 7 only
D. x = 7 and x = -7

Answers

Answer:

[tex] \boxed{C. \: x = 7 \: only} [/tex]

Step-by-step explanation:

[tex] = > {x}^{2} - 14x + 49 = 0 \\ \\ = > {x}^{2} - (7 + 7)x + 49 = 0 \\ \\ = > {x}^{2} - 7x - 7x + 49 = 0 \\ \\ = > x(x - 7) - 7(x - 7) = 0 \\ \\ = > (x - 7)(x - 7) = 0 \\ \\ = > {(x - 7)}^{2} = 0 \\ \\ = > x - 7 = 0 \\ \\ = > x = 7[/tex]

A few years ago, a country consumed 19,400,000 barrels of oil per day. That same year, there were about 119,000,000 households in that country. A barrel of oil is 42 U.S. gallons. Estimate how many gallons each household in the country consumed each day that year by rounding down all three numbers. (Round 19,400,000 barrels to 19,000,000 barrels, 119,000,000 households to 115,000,000 households, and 42 gallons to 40 gallons. Round your answer to two decimal places.)

_________ gal/household/day

Calculate the answer without using estimation and compare the two results. (Round your answer to two decimal places.)
___________gal/household/day

Answers

Following are the calculation to fill in the blank:

For Part a)

Each home in this town consumed

[tex]\to 40 \times \frac{19000000}{115000000}\\\\\to 40 \times \frac{19}{115}\\\\\to 40 \times 0.165\\\\\to 6.6 \ \text{gallons of oil per day}[/tex]

For Part b)

Without completing the sentence,

[tex]\text{Total consumption = 19200000 barrels}\\\\\text{Total number of households = 118000000}[/tex]

As a result, each household consumed.

[tex]\to \frac{19200000}{118000000} \\\\\to \frac{192}{1180} \ (barrels)\\\\[/tex]

In the given scenario, each barrel includes 42 gallons of oil. Therefore, every household consumed [tex]42 \times \frac{192}{1180} = 6.83 \ gallons[/tex] oil each day.

Learn more:

brainly.com/question/14798687

Hence, each household will consume 6.60 gallons per day.

Total Households of that country =115000000 (approx.)

Total barrels used per day =19000000 (approx.)

Given 1 barrel = 40 gallons (approx.)

So total gallons used per day = 19000000 x 40 =760000000

So each household will consume gallons per day = total gallons used per day / Total household of that country

=[tex]=\frac{760000000}{ 11500000}[/tex]

=6.60 gallons/household/day

So each household will consume 6.60 gallons per day.

Learn More: https://brainly.com/question/12272570

PLEASE HELP WILL GIVE BRAINLIEST AND 20 POINTS

solve this system of equations

3y-5x=12
y=1/3x

Answers

Answer:

x=-3

y=-1

(-3,-1)

Answer:

(-3, -1).

Step-by-step explanation:

I have attached the work to your problem.

Please see the attachment below.

I hope this helps!

I need help with is question ASAP please

Answers

The y-intercept looks like it's 0 and the slop looks like 6/2 or 3 or maybe 7/2

Tiffany cells 2 kinds of homemade tomato sauce

Answers

Answer:

b

Step-by-step explanation:

Let x represent the number of quarts of Tuscan sauce and

y represents the number of quarts of marinara sauce Tiffany makes.

A quart of Tuscan sauce requires 6 tomatoes and 1 cup of oil

x quarts requires 6x tomatoes and 1x cups of oil

A quart of her marinara sauce requires 5 tomatoes and 1.25 cups of oil

y quarts requires 5y tomatoes and 1.25 y cups of oil

She has 45 tomatoes and 10 cups of oil on hand.

So the constraints are

6x+5y≤45

1x+1.25y≤10

x>=0 and y>=0

After drawing a diagram of what's required to make 1 quart of Tuscan and 1 quart of Marinara sauce, you can more clearly choose answer choice (B) 6x + 5y < (or equal to) 45, x + 5/4y < (or equal to) 10, x>(or equal to)0, y>(or equal to)0.

please mark brainliest :)

the answer is B !!! hope it’s correct

Which equation represents a line parallel to the line whose equations is -2x + 3y =
-4 and passes through the point (1,3)?

Answers

Answer:

2.  y - 3 = 2/3 (x - 1).

Step-by-step explanation:

-2x + 3y = -4

3y = 2x - 4

y = 2/3 x - 4/3 - so the slope is  2/3.

The slope of a line parallel to it is also 2/3.

It also passes through the point (1, 3).

Using the point-slope form of  a line:

y - y1 = m(x - x1) where m = the slope and (x1, y1) is a point on the line, we have:

y - 3 = 2/3 (x - 1)  <--- is the required equation.

Answer:

2

Step-by-step explanation:

parallel lines have same slope with different intercept

y= mx+b

m is going to be same with different b

so the given function is:

-2x+3y=-43y= 2x-4y= 2/3x - 4/3

Given options:

1. y-3= - 2/3(x-1)  ⇒ y= -2/3x +3 + 2/3 ⇒ y= -2/3x +11/3

it has different slope, so is not parallel

2. y-3= 2/3(x-1) ⇒ y= 2/3x+3-2/3 ⇒ y= 2/3x + 7/3

it has same slope, so is parallelit should be passing through point (1,3)3= 2/3+7/3 ⇒ 3=3, yes it does

3. y-3= -2/3(x+1) ⇒ y= - 2/3x +3- 2/3 ⇒ y= -2/3x + 7/3

it has different slope, is not parallel

4. y-3= 2/3(x+1) ⇒ y= 2/3x +3+ 2/3 ⇒ y= 2/3x +11/3

it has same slope, so is parallelit should be passing through point (1,3)3= 2/3+11/3 ⇒ 3≠13/3, no it doesn't

Helppppp meeeee answer thissssssss

Answers

Answer:

(-1.025)^3 x (-1.025)^2=(- 1.025)^5

(-y)^3 x (-y)^2 x (-y)= y^6

5= odd, - stays

6= even, - becomes +

Answer:

5) (-1.025)^3* (-1.025)^2 = (-1.025)^5 = (-41/40)^5

10) -y^3 * -y^2 * -y

Multiplying an odd number of negative terms makes the product negative

= - y^3 * y^2 * y

= - y^6

14. Roger is on a playground swing, and he is swinging back and forth in such a way that the height, h, in feet, of the swing off the ground is given by the equation h=3cos(3π/2t) +5, where t is in seconds. How many seconds elapses between two consecutive times that the swing is at its maximum height?​

Answers

Answer:

The time [tex]t = \frac{3}{2}[/tex]  seconds elapses between two consecutive times that the swing is at its maximum height  'h' = 2

 Step-by-step explanation:

Explanation:-

Step(i):-

Given function [tex]h(t) = 3 cos (\frac{3\pi }{2 t} ) +5[/tex] ....(i)

By using derivative formulas

[tex]\frac{d cosx }{d x} = -sinx[/tex]

[tex]\frac{d x^{n} }{d x} = n x^{n-1}[/tex]

[tex]\frac{d t^{-1} }{d x} = -1 t^{-1-1} = - t^{-2} = \frac{-1}{t^{2} }[/tex]

Step(ii):-

Differentiating equation(i) with respective to 't'

[tex]h^{l} (t) = 3(-sin(\frac{3\pi }{2t})\frac{d}{dt} (\frac{3\pi }{2t } )+0[/tex]  ...(ii)

[tex]h^{l} (t) = 3(-sin(\frac{3\pi }{2t})(\frac{-3\pi }{2t^{2} } )+0[/tex]

Equating zero

[tex]h^{l} (t) = 3(-sin(\frac{3\pi }{2t})(\frac{-3\pi }{2t^{2} } )=0[/tex]

[tex]3(-sin(\frac{3\pi }{2t})(\frac{-3\pi }{2t^{2} } ) = 0[/tex]

on simplification , we get

[tex](sin(\frac{3\pi }{2t}) = 0[/tex]

now we use formulas

sin 0 = 0 and sinπ = 0

General solution

[tex](sin(\frac{3\pi }{2t}) = sin\pi[/tex]

[tex](\frac{3\pi }{2t}) = \pi[/tex]

Cancellation 'π' on both sides, we get

[tex]3 = 2 t[/tex]

Dividing '2' on both sides , we get

[tex]t = \frac{3}{2}[/tex]

Again differentiating with respective to 't' , we get  

[tex]h^{ll} (t) = 3(-cos(\frac{3\pi }{2t})(\frac{-3\pi }{2t^{2} } )+ (-3)(-sin(\frac{3\pi }{2t} )(\frac{6\pi }{2t^{3} }[/tex]

Put t= 3/2 and simplification

[tex]h^{ll} (t) < 0[/tex]

The maximum height

                   [tex]h(t) = 3 cos (\frac{3\pi }{2 t} ) +5[/tex]

               [tex]h(\frac{3}{2} ) = 3 cos (\frac{3\pi }{2(\frac{3}{2} )} )+5[/tex]

              [tex]h(\frac{3}{2} ) = 3 cos (\pi )+5 = -3+5 =2[/tex]

[tex]t = \frac{3}{2}[/tex]  seconds elapses between two consecutive times that the swing is at its maximum height  'h' = 2

Conclusion:-

The time [tex]t = \frac{3}{2}[/tex]  seconds elapses between two consecutive times that the swing is at its maximum height  'h' = 2

 

 

If g(x) = -4x + 5, find g(2x-1).

Answers

Answer:

- 8x + 9

Step-by-step explanation:

To evaluate g(2x - 1) substitute x = 2x - 1 into g(x), that is

g(2x - 1)

= - 4(2x - 1) + 5 ← distribute parenthesis and simplify

= - 8x + 4 + 5

= - 8x + 9

Answer:

= - 8x + 9

Step-by-step explanation:

= - 8x + 4 + 5

= - 8x + 9

please help as soon as possible
I WILL MARK YOU AS BRAINLIEST ​

Answers

Answer:

Last choice

Step-by-step explanation: Subtract 11 and then divide by 3. A would be greater than -2 and last choice shows that

Arrange from smallest to largest
0.01, 1.01, 10.01, 1.001

Answers

Answer:

0.01 --> 1.001 ---> 1.01 ---> 10.01

The numbers in the ascending order:

0.01, 1.01, 1.001, 10.01.

What is ascending order?

Putting numbers in ascending order simply means to do it from smallest to largest.

Given:

0.01, 1.01, 10.01, 1.001.

The numbers from smallest to largest:

That means, in the ascending order.

So,

0.01, 1.01, 1.001, 10.01.

Therefore, 0.01, 1.01, 1.001, 10.01 are in the required order.

To learn more about the ascending order;

https://brainly.com/question/20681445

#SPJ2



A new car is purchased for 23400 dollars. The value of the car depreciates at 11.5% per year. To the nearest year, how long will it be until the value of the car is 12700 dollars?

Answers

Answer: 5 years

Step-by-step explanation:

Using same formula as for Compound Interest:

12700 = 23400 (1-.115)^t

12700 = 23400 x 0.885^t

0.885^t = 127/234

Convert decimal into fraction:

177/200^t = 127/234

Take logarithm of both sides of thr equation:

t = log 177/200 (127/234)

t = 5.00242


[tex]( {2}^{ - 1} + {3}^{ - 1} )^{2} [/tex]
solve.

will give the brainliest​

Answers

Answer:

25/36

Step-by-step explanation:

(2^-1 + 3^-1)^2

(1/2 + 1/3)^2

(5/6)^2 = 25/36

25/36 or .694

2^-1= 1/2
3^-1=1/3

Add the together
5/6 square it and you get 25/36

Please Help me it’s geometry

Answers

Answer:

Option C.

Step-by-step explanation:

Given: In [tex]\Delta OPQ,m\angle O=107^{\circ},m\angle P=28^{\circ} [/tex].

In [tex]\Delta OPQ[/tex],

[tex]m\angle O+m\angle P+m\angle Q=180^{\circ}[/tex]    (Angle sum property)

[tex]107^{\circ}+28^{\circ}+m\angle Q=180^{\circ}[/tex]

[tex]135^{\circ}+m\angle Q=180^{\circ}[/tex]

[tex]m\angle Q=180^{\circ}-135^{\circ}[/tex]

[tex]m\angle Q=45^{\circ}[/tex]

Now,

[tex]m\angle O>m\angle Q>m\angle P[/tex]

In a triangle, the greatest angle has largest opposite side and smallest angle has smallest opposite side. So, we conclude that

[tex]PQ>OP>QO[/tex]

Therefore, the correct option is C.

Other Questions
Choose the adverbial phrase in the sentence.The boy in the blue raincoat hung his hat on the coat rack.A. on the coat rackB. The boyC. in the blue raincoatD. hung his hat Based on this information, what can you conclude about how the narrator has spent her time over the years? Does it seem like the narrator let go of her bitterness about losing her job? Support your answer using information from the passage. Make sentences to show necessity, request, suggestion using- must, should, would Which of these is a disadvantage of hydroelectric energy ? A. Is expensive to build or maintain B. Has high power plant efficiency C. Has a long power plant life span D. Reduces greenhouse gas emissions What type of study would you conduct if you wanted to know how manymiles a week on average students on the school's cross country team runduring the summer? Explain your reasoning. *Your answer any ideas of a specific type of dog What is the slope of this line? 4x + 5y = 7 3x 2y = 12 Multiply each equation by a number that produces opposite coefficents for x or y Give the correct form of ser or estar to complete each sentence.1. La Sra. Sosa ___ de la Repblica Dominicana.2. Su casa ___ en Santo Domingo pero ahora ella en Espaa con sus hijos.3. Ella ___ inteligente y sus hijos altos.4. Sus hijos ___ contentos porque van a un concierto.5. El concierto ___ en el parque. How does Wheeler describe Jim smiley Compare the mass of the two objects.Which statement accurately describes the mass of the watermelon?The mass of the watermelon is less than 2 kg.The mass of the watermelon is equal to 2 kg.The mass of the watermelon is greater than 2 kg. PLEASE HELP ME QUICK EXPLANATION AND ANSWERS PLEASE IF ANY NONSENSE ANSWED YOU WILL BE REPORTED The party is this Friday night fortunately my parents have given permission for me togo.How should this sentence be punctuated? 100 mL of a buffer that consists of 0.20 M NH3 and 0.20 M NH4Cl is titrated with 25 mL of 0.20 M HCl. Calculate the pH of the resulting solution given that the Kb for NH3 is 1.8 x 10-5. Work out the size of angle x Complete the sentence below correctly.A compression of a longitudinal wave is like aof a transverse wave.OA) crestOB) troughOC) rarefactionOD) compression Yo,_______(tener) miedo que mi abuelo_______(estar)enfermo.O tenga, estO tengo , estO tenga, est A boy pushes on a wagon so that it accelerates at a rate of 0.50 m/s2. The wagon has a mass of 24 kg. What is the magnitude of the boy's pushing force? (Ignore frictional effects.) 12 N 12 kg 48 N 48 kg 9. What is the volume of the triangular prismbelow?5cm3 cm4cm3 cmNeed help ASAP A coin is tossed then a letter from the word incredible is chosen at random. What is the probability of getting heads, then a vowel?